¿Permanece en la mecánica cuántica el 4343\frac{4}{3} problema del electromagnetismo clásico?

En el Volumen II Capítulo 28 de las Conferencias Feymann sobre física, Feynman analiza el infame 4 3 problema del electromagnetismo clásico. Suponga que tiene una partícula cargada de radio a y carga q (distribuido uniformemente en la superficie). Si integra la densidad de energía del campo electromagnético en todo el espacio fuera de la partícula, obtendrá la energía electromagnética total, que es una expresión proporcional a C 2 . La energía dividida por C 2 es lo que solemos llamar la masa, por lo que si calculamos la "masa electromagnética" de esta manera obtendremos metro = 1 2 1 4 π ϵ 0 q 2 a C 2 . Si, por otro lado, tomaras la densidad de momento del campo electromagnético y la integraras en todo el espacio fuera de la partícula, obtendrías el momento electromagnético total, que resulta (por v << C ) ser proporcional a la velocidad de la partícula. La constante de proporcionalidad del momento y la velocidad es lo que llamamos masa, por lo que si calculáramos la masa electromagnética de esta forma obtendríamos metro = 2 3 1 4 π ϵ 0 q 2 a C 2 , cual es 4 3 veces el valor que obtuvimos antes! Eso es el 4 3 problema.

Feynman afirma que este problema fundamental permanece cuando pasamos a la electrodinámica cuántica. ¿Tenía razón, y si es así ha cambiado la situación desde la 1960 s cuando estaba escribiendo? He visto afirmaciones en Internet (no tengo los enlaces) de que el 4 3 el problema sigue ahí en QED, pero en lugar de 4 3 el coeficiente es algo más cercano a 1. ¿Es eso cierto, y si es así, cuál es el coeficiente? Todo esto, por supuesto, está relacionado con cuestiones de autoenergía y renormalización.

Cualquier ayuda sería muy apreciada.

Respuestas (4)

Esquema del problema

En la referencia citada, Feynman comienza su argumento afirmando que la energía de una esfera de inercia de radio a y carga uniforme q es dado por

tu mi yo mi C = R 3 d V tu mi yo mi C = R 3 d V ( ϵ 0 mi 2 ) = 1 2 a q 2 4 π ϵ 0 mi 2 2 a ( )

dónde mi y tu mi yo mi C son su campo eléctrico y densidad y la última ecuación define el símbolo mi 2 . Si ahora consideramos que la esfera se mueve con velocidad constante v C , su densidad de momento se obtiene del vector de Poynting:

pags = R 3 d V S = R 3 d V ( ϵ 0 mi × B ) = ( 2 3 mi 2 a C 2 ) v metro mi v , ( )

donde la última ecuación define la masa electromagnética metro mi . También podemos asociar una segunda masa eléctrica metro mi Al campo tu mi yo mi C usando la relatividad especial (SR):

mi = metro C 2 tu mi yo mi C C 2 = metro mi mi 2 2 a C 2 = metro mi = 3 4 metro mi tu mi yo mi C = 4 3 metro mi C 2 ,

es decir, la masa "relativista" metro mi no es lo mismo que la masa "electrica" metro mi , siendo el absurdo explícito en las ecuaciones anteriores.

la escapatoria

¿Tenemos derecho a invocar SR en un solo paso de derivación? Aunque usamos la famosa fórmula de Einstein para llegar a la 4 3 problema, observe que no hemos usado SR antes: ( ) y ( ) ¡Ni siquiera son invariantes de Lorentz! Si adaptamos los cálculos para incluir SR, tu mi yo mi C y pags transformar como

tu mi yo mi C = γ ( tu mi yo mi C + pags v ) pags = γ ( pags + tu mi yo mi C v C 2 ) + ( γ 1 ) v × ( v × pags ) .

Es decir, para una partícula en reposo tenemos pags = 0 e, introduciendo la velocidad de una manera relativistamente significativa al cambiar a un marco de referencia móvil con velocidad v ,

tu mi yo mi C = γ tu mi yo mi C = γ ( mi 2 2 a ) = γ metro mi C 2 pags = γ ( tu mi yo mi C C 2 ) v = γ ( mi 2 2 a C 2 ) v = γ metro mi v ,

donde ahora, he aquí, todo encaja y no hay absolutamente nada 4 3 paradoja. Solo era cuestión de moverse correctamente entre los marcos de referencia.

¿Qué quiso decir Feynman?

Siendo uno de los físicos más brillantes de la historia, Richard Feynman nunca abusaría de la RS como si fuera necesario para llegar a la 4 3 paradoja. De hecho, Abraham y Lorentz, los descubridores, solo lo hicieron porque no sabían realmente cómo transformar entre marcos de referencia, ya que SR no se había terminado en ese momento. Citemos entonces lo que realmente dijo Feynman:

Hay dificultades asociadas con las ideas de la teoría de Maxwell que no se resuelven ni se asocian directamente con la mecánica cuántica. Puedes decir: “Quizás no tenga sentido preocuparse por estas dificultades. Dado que la mecánica cuántica va a cambiar las leyes de la electrodinámica, deberíamos esperar a ver qué dificultades hay después de la modificación”. Sin embargo, cuando se une el electromagnetismo a la mecánica cuántica, las dificultades persisten.

De hecho, el 4 3 El problema no se resuelve mediante la mecánica cuántica (QM), sino mediante el uso cuidadoso de SR. Abraham y Lorentz usaron ( ) y ( ) indiscriminadamente porque no sabían que los electrones en movimiento se deforman en elipsoides según SR. El hermoso artículo de Rohrlich , del cual esta respuesta es un resumen, muestra que el error al ignorar la deformación del electrón equivale exactamente a 1 3 metro mi C 2 , lo que corregiría su resultado y también eliminaría el 4 3 problema. Creo que Feynman estaba hablando de otros problemas que migran de la electrodinámica clásica a la cuántica (QED), no de esta aparente paradoja. El problema de la autoenergía , por ejemplo, requiere que el electrón en QED tenga una estructura interna poco intuitiva o interactúe con cosas que tienen masa negativa, de lo contrario explotaría (aparte de la renormalización). Este problema migra erróneamente de la electrodinámica clásica debido a la postulación de un "estrés de Poincaré" que no solo estabilizaría al electrón, sino que también corregiría el 1 3 factor que falta en el 4 3 problema. Afortunadamente, muchos físicos ayudaron a disociar el problema de la estabilidad electrónica de la 4 3 problema, que en realidad no es un problema, ya que en realidad no aparece en QED. Sin embargo, el problema de la estructura interna del electrón es bastante real.

"¿Tenemos el derecho de invocar SR en un solo paso de derivación? Aunque usamos la famosa fórmula de Einstein para llegar al problema 43, observe que no hemos usado SR antes: (∗) y (∗∗) ni siquiera son invariantes de Lorentz !" En realidad, Feynman tiene totalmente en cuenta el problema de la deformación, pero no resuelve el problema del 4/3. Desplácese hacia abajo hasta donde dice "Sigamos con nuestra teoría electromagnética de la masa. Nuestro cálculo fue para v≪c; ¿qué sucede si vamos a altas velocidades?" feynmanlectures.caltech.edu/II_28.html Obtienes una forma invariante de Lorentz pero aún tiene un 2323 factor.
Primero, la fórmula 28.7 de Feynman es para el cambio relativista de la masa incorrecta, m emetromi , que no surge en SR: El término correcto es m emetromi . En segundo lugar, tener en cuenta por completo la deformación de los electrones implica no descuidar una integral que solo es nula dentro de un electrón desnudo. Le insto a que consulte el artículo de Rohrlich, donde todo está extremadamente bien presentado, especialmente la última sección donde resuelve el tensor de energía de tensión en el interior y el exterior de un electrón elipsoidal.
No sé qué quieres decir con "la fórmula 28.7 de Feynman es para el cambio relativista de la masa incorrecta, m emetromi , que no surge en SR". La fórmula 28.7 solo está calculando el momento relativista, todavía no se trata de la masa electromagnética.
Verifique de cerca: Feynman define la masa electromagnética m emetromi en 28,4, con el factor 2/3 delante. Luego lo usa más tarde en 28.7 para definir el momento relativista. Sin embargo, esto no está bien, y Feynman solo está calculando las consecuencias de una mala elección. La masa propia y correcta proviene de SR y no tiene factor 2/3 frente a ella, ya que está dada por m e = U e l e c / c 2metromi=tueléctrico _ _ _/C2 . Esta masa, entonces, se usa más tarde para obtener el momento relativistamente correcto p = γ m e vpags⃗ = γmetromiv⃗  . La masa en 28.4 es incorrecta , ya que proviene de una expresión que no es correcta en SR.
No deriva 28.7 de 28.4. Primero deriva 28,4 de 28,3, luego proporciona 28,7 como el análogo relativista del cálculo realizado para obtener 28,3, para mostrar que 28,4 proviene de 28,7 de la misma manera que proviene de 28,3.
Reforzaré lo que dije antes: la masa propia que aparece cuando se usa el momento relativista correcto es m emetromi , no yometromi . Esto se hace explícitamente en mis cálculos. La fórmula de Feynman 28.7 es incorrecta .
Eso no puede ser un problema con la fórmula 28.7, porque la fórmula 28.7 no se trata de masa electromagnética en absoluto. Se trata del momento electromagnético, y luego se usa 28.7 para obtener la masa electromagnética.
Ejercicio 1: Llegar a la fórmula 28.7 sin partir de un momento inicial igual a m e vmetromiv⃗  . Pista: es imposible. Ejercicio 2: define la masa usando m e = U e l e c / c 2metromi=tueléctrico _ _ _/C2 , luego rehaga los cálculos que mostré y llegue al momento correcto en el régimen de velocidad pequeño m e vmetromiv⃗  .
Feynman describe a grandes rasgos cómo se hace, y no es usando la fórmula 28.4. Es integrando la densidad del momento teniendo en cuenta la relatividad: "Los primeros intentos generaron cierta confusión, pero Lorentz se dio cuenta de que la esfera cargada se contraería en un elipsoide a altas velocidades y que los campos cambiarían de acuerdo con las fórmulas (26.6 ) y (26.7) que derivamos para el caso relativista en el Capítulo 26. Si lleva a cabo las integrales para p en ese caso..." Dudo mucho que Feynman estuviera fanfarroneando, presumiblemente hizo el cálculo relevante.
Te dejaré hacer los ejercicios que te propuse. Entonces, por favor, encuentra el error en: for p = 0pags⃗ = 0 , metro mi = U mi l mi C / C 2pags = γ U mi l mi C v / C 2 = γ metro mi vmetromi=tueléctrico _ _ _/C2pags⃗ = γtueléctrico _ _ _v⃗ /C2= γmetromiv⃗  . Este cálculo fue realizado por Schwinger, Fermi, Rohrlich y docenas de otros. Sin comprender mi respuesta correctamente o leer las referencias que proporcioné, no tiene sentido continuar esta discusión.
¿Por qué no hago esto? Puedo publicar una nueva pregunta en el sitio preguntando cómo se hace la integral para la fórmula 28.7. Como dijiste, Feynman es "uno de los físicos más brillantes de la historia", por lo que no debemos descartar lo que dice sobre la integración para encontrar p .pags fuera de alcance.
Puedes preguntar esto, pero es trivial: para llegar a 28.7, Feynman partió de 28.3 y luego usó la ley de transformación p = γ ppags⃗ = γpags⃗  empezar desde el impulso inicial m e vmetromiv⃗  y asignarlo a γ m e vγmetromiv⃗  . Evidentemente , la ecuación 28.7 no es relativistamente correcta, ya que el 2/32 / 3 el factor frente a él evita que el 4-momentum asociado se transforme en un 4-vector. Por lo tanto, la ecuación 28.7 usa la ley de transformación correcta para la cantidad de movimiento incorrecta, obtenida a partir de una ecuación relativistamente incorrecta. La correcta se obtiene de p = m e vpags⃗ =metromiv⃗  .
"para llegar a 28.7 Feynman partió de 28.3, y luego usó la ley de transformación p = γ p para empezar desde el impulso inicial m e v y asignarlo a γ m e v ." Eso no es lo que dice que hizo. Lo que dice que hizo es integrar la densidad de cantidad de movimiento, utilizando las transformaciones de campo de las fórmulas 26.6 y 26.7.

En la visión moderna de la teoría cuántica de campos, la 4 / 3 El problema, así como las cuestiones relacionadas con los cargos puntuales, son cuestiones de regularización.

El problema es que quieres tratar con una cantidad singular, a saber, la energía y el momento de una carga puntual ideal, pero es imposible decir algo definitivo porque todas estas cantidades son divergentes. Para obtener una respuesta finita, modifica el sistema regularizándolo de alguna manera, por lo que las respuestas resultan finitas. En general, la regularización puede y romperá las simetrías, y generalmente hacemos todo lo posible para elegir un esquema de regularización que conserve las simetrías que más nos interesan.

La razón filosófica por la que esto funciona es porque sabemos que nuestras teorías solo son válidas hasta una determinada escala de corte. Λ , por encima de la cual aparecen nuevos fenómenos. Por debajo del límite, los efectos de estos nuevos fenómenos se pueden parametrizar en términos de un Lagrangiano efectivo. Por lo tanto, podemos obtener las mismas predicciones de baja energía en cualquier esquema de regularización que produzca el mismo Lagrangiano efectivo, incluso si estos esquemas colocan física dramáticamente diferente por encima de la escala de corte.

En este caso, Feynman está regularizando la teoría clásica cortando las integrales de campo en Λ 1 / a . Desafortunadamente, este esquema de regularización no preserva la invariancia de Lorentz y, además, la invariancia de Lorentz no se restaura ni siquiera en el límite continuo. Λ . Esto no es algo raro. Por ejemplo, la QCD de celosía también tiene que abordar este problema, porque una regularización de celosía rompe claramente la invariancia de Lorentz.

La razón por la que la red QCD funciona de todos modos, en el lenguaje RG, es que sus sistemas están diseñados para que solo se puedan producir operadores irrelevantes adicionales, cuyos efectos se desvanecen en el límite continuo. a 0 . Intuitivamente, el problema con la regularización de Feynman es que permite operadores extra relevantes, que estropean el Lagrangiano efectivo sin importar qué. a es, aunque este lenguaje no es exacto ya que Feynman no se trata de una teoría cuántica de campos.

La resolución moderna de la 4 / 3 problema en la teoría cuántica de campos es esencialmente por decreto. Una teoría no regularizada está mal definida; no podemos hablar de QED no regularizado porque tal teoría ni siquiera existe. En cambio, necesitamos un regulador en mente desde el principio. Como sabemos que la invariancia de Lorentz es una simetría de nuestro mundo, elegimos un regulador que recupere la invariancia de Lorentz en el límite continuo.

Me doy cuenta de que el problema surgió originalmente al comprender la física de las partículas cargadas puntuales y reemplazar la partícula cargada puntual con una capa cargada es una forma muy física de poner solo la regularización de corte que rompe la simetría de Lorentz. Sin embargo, uno puede olvidarse de la cuestión más importante de la física de las partículas cargadas puntuales y estar simplemente interesado en la física de una capa cargada por derecho propio. Si uno lo hace, creo que el 4 / 3 problema sigue siendo una pregunta válida para hacer, ¿verdad?

Aparte de H. Poincaré, el primer físico que hizo una "derivación" relativista correcta fue E. Fermi.

Aquí me gustaría centrarme en el efecto físico de la famosa autoacción: es simplemente una autoinducción que "ralentiza" o "resiste" a cualquier aceleración (cambio en el tiempo de una corriente constante). Aparentemente, no es un efecto deseado, no es una pequeña resistencia radiativa en absoluto, sea cual sea el tamaño del electrón.

Tenga en cuenta que el EMF "se mueve" bien con el electrón, de acuerdo con la ecuación de Maxwell, por lo que no es necesario tenerlo en cuenta "nuevamente" en las ecuaciones electrónicas (mecánicas). La resistencia radiativa debe adivinarse a partir de diferentes ideas físicas en lugar de "auto-acción".

En QED, este problema permanece y se "resuelve" modificando las soluciones de ecuación (incorrectas) en lugar de modificar las ecuaciones incorrectas.

Los resultados de mi investigación aún están en un estado embrionario, pero procedo de otra idea física que capta bien la resistencia radiativa.

Estoy de acuerdo con "En QED, este problema permanece y se 'resuelve' modificando las soluciones de ecuación (incorrectas) en lugar de modificar las ecuaciones incorrectas". Desafortunadamente, esta respuesta es en gran medida incomprensible.
@QuantumBrick: Lástima. Tengo otra referencia a Rohlich: Fritz Rohrlich, La dinámica de una partícula cargada, (2008) arxiv.org/abs/0804.4614 donde afirma que en CED el problema aún no está resuelto (de manera satisfactoria).

Con respecto al problema descrito en el Volumen II Capítulo 28 de las Lecciones de Física de Feynman, la razón del problema 4/3 es que uno de los términos que afectan el movimiento de la partícula no se tiene en cuenta. Ese término es la destrucción del campo de energía en el frente de la esfera cargada y su creación en la parte de atrás. Si bien la energía general del campo no está cambiando, si evalúa la energía localmente, puede ver que en el frente, el término v·E significa que la energía se lleva del campo a la partícula y en la parte posterior, sucede lo contrario, como esto sucede todo el tiempo su efecto es equivalente a un movimiento hacia atrás. Cuando lo agrega al movimiento normal causado por el impulso, descubre que necesita una masa m_e = (1/2)e^2/a y no hay más inconsistencias.

Coloqué un pequeño documento sobre el tema aquí: https://www.slideshare.net/SergioPL81/adding-a-shift-term-to-solve-the-43-problem-in-classical-electrodinamics